A 1,760 W toaster, a 1,420 W electric frying pan, and an 85 W lamp are plugged into the same outlet in a 15 A, 120 V circuit. (The three devices are in parallel when plugged into the same socket.) (a) What current (in A) is drawn by each device

Answers

Answer 1

Answer:

Toaster = I = 14.67 A

Frying Pan = 11.83 A

Lamp = 0.71 A

Explanation:

The electric power is given as:

[tex]P = VI\\\\I = \frac{P}{V}[/tex]

where,

I = current

P = Power

V = Voltage = 120 V

FOR TOASTER:

P = 1760 W

Therefore,

[tex]I = \frac{1760\ W}{120\ V}[/tex]

I = 14.67 A

FOR FRYING PAN:

P = 1420 W

Therefore,

[tex]I = \frac{1420\ W}{120\ V}[/tex]

I = 11.83 A

FOR LAMP:

P = 85 W

Therefore,

[tex]I = \frac{85\ W}{120\ V}[/tex]

I = 0.71 A


Related Questions

1. A sequence of potential differences v is applied accross a wire (diameter =0.32 mm length = 11 cm and the resulting current I are measured as follows: V 0.1 0.2 0.3 0.4 0.5 I (MA) 72 144 216 288 360 2) a) plot a graph of v against I.
b) determine the wire's resistence , R.
c) State ohm's law and try to relate it . your results.​

Answers

Answer:

a. Find the graph in the attachment

b. 720 kΩ

c. The ratio V/I gives us our resistance which is 720 kΩ

Explanation:

a) plot a graph of V against I.

To plot the graph of V against I, we plot the corresponding points against each other. With the voltage V measured in volts and the current I measured in mA, the plotted graph is in the attachment.

b) Determine the wire's resistance , R.

The resistance of the wire is determined as the gradient of the graph.

R = ΔV/ΔI = (V₂ - V₁)/(I₂ - I₁)

Taking the first two corresponding measurements. V₁ = 72 V, I₁ = 0.1 mA, V₂ = 144 V and I₂ = 0.2 mA

R = (144 V - 72 V)/(0.2 - 0.1) mA

R = 72 V/0.1 mA

R = 72 V/(0.1 × 10⁻³ A)

R = 720 × 10³ V/A

R = 720 kΩ

c) State ohm's law and try to relate it your results.​

Ohm's law states that the current flowing through a conductor is directly proportional to the voltage across it provided the temperature and all other physical conditions remain constant.

Mathematically, V ∝ I

V = kI

V/I = k = R

Since the ratio V/I = constant, from our results, the ratio of V/I for each reading gives us the resistance. Since we have a linear relationship between V and I, the gradient of the graph is constant and for each value of V and I, the ratio V/I is constant. So, the ratio V/I gives us our resistance which is 720 kΩ.

Since V/I is constant, we thus verify Ohm's law.

describe four energy changes that happen in the process.

Answers

Driving a motor........

chemical energy is converted into kinetic energy.

Falling off of cliff

.........gravitational potential energy is converted into kinetic energy.

Hydroelectric energy generation

.......gravitational potential energy is converted into kinetic energy (i.e. driving a generator), which is then converted into electrical energy.

Nuclear power generation

.........mass is converted into energy, which then drives a steam turbine, which is then converted into electrical energy.

how will be electric lines of force where intensity of electric field is maximum ?
a. wider
b. +ve to -ve
c. narrow
d. -ve to +ve

Answers

i'm pretty sure the answer is A wider

Electric lines of force where intensity of electric field is maximum when its wider.

What is Electric field?

The physical field that surrounds electrically charged particles and exerts force on all other charged particles in the field, either attracting or repelling them, is known as an electric field (also known as an E-field. It can also refer to a system of charged particles' physical field.

Electric charges and time-varying electric currents are the building blocks of electric fields. The electromagnetic field, one of the four fundamental interactions (also known as forces) of nature, manifests itself in both electric and magnetic fields.

Electrical technology makes use of electric fields, which are significant in many branches of physics. For instance, in atomic physics and chemistry, the electric field acts as an attracting force to hold atoms' atomic nuclei and electrons together.

Therefore, Electric lines of force where intensity of electric field is maximum when its wider.

To learn more about electric field, refer to the link:

https://brainly.com/question/1443103

#SPJ2

An airplane which intends to fly due south at 250 km/hr experiences a wind blowing westward at 40 km/hr. What is the actual speed of the airplane relative to the ground? ​

Answers

Answer:

simple is rumple a daily ok I'll be

what is the energy of an electromagnetic wave that has a frequency of 8.0 x 10^15 Hz? Use the equation...

Answers

(C)

Explanation:

[tex]E = hf = (6.626×10^{-34}\:\text{J•s})(8.0×10^{15}\:\text{Hz})[/tex]

[tex]= 5.3×10^{-18}\:\text{J}[/tex]

Answer:

It's D

Explanation:

It's from alvs

A 2.5 kg block slides along a frictionless surface at 1.5 m/s.A second block, sliding at a faster 4.1 m/s , collides with the first from behind and sticks to it. The final velocity of the combined blocks is 2.5 m/s. What was the mass of the second block?

Answers

Answer:

1.5kg

Explanation:

Given data

mass m1= 2.5kg

mass m2=??

velocity of mass one v1= 1.5m/s

velocity of mass two v2= 4.1m/s

common velocity after impact v= 2.5m/s

Let us apply the formula for the conservation of linear momentum for inelastic collision

The expression is given as

m1v1+ m2v2= v(m1+m2)

substitute

2.5*1.5+ m2*4.1= 2.5(2.5+m2)

3.75+4.1m2= 6.25+2.5m2

collect like terms

3.75-6.25= 2.5m2-4.1m2

-2.5= -1.6m2

divide both sides by -1.6

m2= -2.5/-1.6

m2= 1.5 kg

Hence the second mass is 1.5kg

) The velocity function is v(t)=−t2+3t−2v(t)=−t2+3t−2 for a particle moving along a line. Find the displacement (net distance covered) of the particle during the time interval [−2,5][−2,5].

Answers

Answer:

89.87m/s

Explanation:

Given the velocity function

v(t)=−t²+3t−2

In order to get the displacement function, we will integrate the velocity function as shown:

[tex]\int\limits^5_{-2} {v(t)} \, dt \\d(t)= \int\limits^5_{-2}{(-t^2+3t+2)} \, dt \\\\d(t)=[\frac{-t^3}{3}+\frac{3t^2}{2}+2t ]^5_{-2}\\[/tex]

at t = 5

[tex]d(5)=[\frac{-5^3}{3}+\frac{3(5)^2}{2}+2(5) ]\\d(5)=[\frac{-125}{3}+\frac{75}{2}+10 ]\\d(5)=-41.7+37.5+10\\d(5)=89.2m/s[/tex]

at t = -2

[tex]d(-2)=[\frac{-(-2)^3}{3}+\frac{3(-2)^2}{2}+2(-2) ]\\d(-2)=[\frac{-8}{3}+\frac{12}{2}+(-4) ]\\d(-2)=-2.67+6-4\\d(-2)=-0.67m/s[/tex]

Required displacement = d(5) - d(-2)

Required displacement = 89.2 - (-0.67)

Required displacement = 89.2 + 0.67

Required displacement = 89.87m/s

A roller coaster uses 800 000 J of energy to get to the top of the first hill. During this climb, it gains 500 000 J of potential energy and pauses (velocity = 0) for a fraction of a second at the very top before heading down the other side.

a) Draw a sankey diagram for a roller coaster's climb.

Answers

A roller coaster uses 800 000 J of energy to get to the top of the first hill. During this climb, it gains 500 000 J of potential energy and pauses for a fraction of a second at the very top before heading down the other side. At the top of the hill total, the kinetic energy of the roller coaster would be zero as the velocity is zero at the top of the hill, therefore the total mechanical energy is only because of potential energy.

What is mechanical energy?

Mechanical energy is the combination of all the energy in motion represented by total kinetic energy and the total stored energy in the system which is represented by total potential energy.

The expression for total mechanical energy is as follows

ME= KE+PE

As total mechanical energy is the sum of all the kinetic as well as potential energy stored in the system.As given in the problem a roller coaster uses 800000 J of energy to get to the top of the first hill. During this climb, it gains 500 000 J of potential energy which means 300000 J of energy is lost in the frictional energy while climbing the hill,

Thus at the top of the hill, the total energy of the roller coasters is only due to the potential energy.

Learn more about mechanical energy from here brainly.com/question/12319302

#SPJ2

A roller coaster has a mass of 1200.0kg. The coaster is going 22.0 m/s at the bottom
of the third loop-the-loop that is 2.5m above the ground. Determine the height of
the first hill that is required, assuming the cart is stationary at the top of the first hill
before it falls.

Answers

Answer:

h = 27.17 m

Explanation:

First, we will calculate the total mechanical energy of the system at the bottom point of the third loop:

Mechanical Energy = Kinetic Energy + Potential Energy

[tex]E = \frac{1}{2}mv^2 + mgh[/tex]

where,

E = Total Mechanical Energy = ?

m = mass of the roller coaster = 1200 kg

v = velocity of the roller coaster = 22 m/s

g = acceleration due to gravity = 9.81 m/s²

h = height of roller coaster = 2.5 m

Therefore,

[tex]E = \frac{1}{2}(1200\ kg)(22\ m/s)^2+(1200\ kg)(9.81\ m/s^2)(2.5\ m)\\\\E = 290400 J +29430\ J\\\\E = 319830\ J = 319.83\ KJ[/tex]

Now, the total mechanical energy at the top position of the first hill must also be the same:

[tex]E = \frac{1}{2}mv^2 + mgh[/tex]

where,

v = 0 m/s

h = ?

Therefore,

[tex]319830\ J = \frac{1}{2}(1200\ kg)(0\ m/s)^2+(1200\ kg)(9.81\ m/s^2)(h)\\\\h = \frac{319830\ J}{11772\ N}\\\\[/tex]

h = 27.17 m

A particle moves along the x axis. In order to calculate the torque on the particle, you need to know:

a. the rotational inertia of the particle
b. the velocity of the particle
c. the mass of the particle
d. the kinetic energy of the particle
e. the point about which the torque is to be calculated

Answers

Answer:

e. the point about which the torque is to be calculated

Explanation:

torque is the product of a force and a distance

the point about which the torque is calculated is required to know the distance.

None of the other terms are relevant as they refer to mass or its equivalent, and velocity. Force is not mentioned in any of them.

A particle moves along the x-axis. In order to calculate the torque on the particle, you need to know the point about which the torque is to be calculated. Therefore, option E is correct.

What is torque ?

The rotating equivalent of force is torque. Depending on the subject of study, it is also known as the moment, moment of force, rotating force, or turning effect. It illustrates how a force can cause a change in the body's rotational motion.

Ancient Romans gave these necklaces the term "torque" by describing them as twisted and spiral screw-shaped using the Latin word "torquere," which also means "twisting" and "turning."

It's critical to realize that torque, which has to do with your motor's power in terms of rotational force, is not the same thing as speed. Find a motor with a top speed if you require more motor speed, and a motor with a motor torque that is maximized if you need more rotational force.

Thus, option E is correct.

To learn more about torque, follow the link;

https://brainly.com/question/9270821

#SPJ2

A sports car accelerates uniformly from rest to 24 m/s in 6 seconds. Calculate the acceleration of the car

Answers

Answer:

a = 4m/s^2

Explanation:

Velocity(V) = uniform = 24m/s

time(t) = 6sec

Acceleration(a) = V/t

= 24/6

= 4m/s^2

When a sports car accelerates uniformly from rest to 24 m/s in 6 seconds,then acceleration of the car would be 4 m/s²

What are the three equations of motion?

There are three equations of motion given by  Newton

The first equation is given as follows

v = u + at

the second equation is given as follows

S = ut + 1/2×a×t²

the third equation is given as follows

v² - u² = 2×a×s

Note that these equations are only valid for a uniform acceleration.


As given problem sport car accelerates uniformly from rest to 24 m/s in 6 seconds then the acceleration of the car can be calculated by using the first equation of motion

v = u + at

As given the initial velocity u= 0

The final velocity v = 24 m/s

The time taken is t= 6 seconds

By substituting the respective values of velocity and  time

24 = 0+ a*6

a = 24/6

a = 4 m/s²

Thus, when a sports car accelerates uniformly from rest to 24 m/s in 6 seconds,then acceleration of the car comes out to be 4 m/s²

Learn more about  equations of motion from here

brainly.com/question/5955789

#SPJ2

When you shine a beam of light, which is composed of just two different colors, red and green, onto a diffraction grating which color gets diffracted more

Answers

Answer:

The diffraction grating separates light into colors as the light passes through the many fine slits of the grating. This is a transmission grating. ... The prism separates light into colors because each color passes through the prism at a different speed and angle.

Light of frequency f falls on a metal surface and ejects electrons of maximum kinetic energy K by the photoelectric effect. If the frequency of this light is doubled, the maximum kinetic energy of the emitted electrons will be

Answers

The question is incomplete, the complete question is;

Light of frequency f falls on a metal surface and ejects electrons of maximum kinetic energy K by the photoelectric effect.

Part A If the frequency of this light is doubled, the maximum kinetic energy of the emitted electrons will be If the frequency of this light is doubled, the maximum kinetic energy of the emitted electrons will be

K/2.

K.

2K.

greater than 2K.

Answer:

2K

Explanation:

Given that the kinetic energy of photo electrons is given by;

K= E -Wo

Where;

K = kinetic energy

E= energy of incident photon

Wo = work function

But;

E= hf

Wo = fo

h= Plank's constant

f= frequency of incident photon

fo= Threshold frequency

So:

K= hf - hfo

Where the frequency of incident light is doubled;

K= 2hf - hfo

Hence, maximum kinetic energy of the emitted electrons in this case will be 2K

Consider a piston filled with 3 mols of an ideal gas, kept at a constant temperature 290 K. We slowly compress the gas starting at 2 m3 and ending at 1 m3. How much work do we need to do on the gas to perform this operation

Answers

Answer: [tex]-5013.65\ J[/tex]

Explanation:

Given

No of moles [tex]n=3[/tex]

Temperature [tex]T=290\ K[/tex]

Initial volume [tex]V_1=2\ m^3[/tex]

Final volume [tex]V_2=1\ m^3[/tex]

Work done in constant temperature process is

[tex]W=nRT\ln \left(\dfrac{V_2}{V_1}\right)[/tex]

Insert the values

[tex]\Rightarrow W=3\times 8.314\times 290\ln \left (\dfrac{1}{2}\right)\\\\\Rightarrow W=-870\times 8.314\times \ln (2)\\\Rightarrow W=-5013.65\ J[/tex]

Why do we use semiconductor instead of metal in thermopile?

Answers

Answer:

Metal or conductors , what they do is that they allow full flow of current that is conduction is due to free electrons only and there is literally no gap between valence and conduction band,so free electrons can easily jump into conduction band from valence band.

And now lets talk about insulators,what they do is that they don’t allow any current to flow i.e they act as strong dielectric,and gap between valence band and conduction band is so big that free electron can never come into conduction band from valence band ever if they try is for eternity…

But semiconductors can act both as an insulator as well as a conductor based on the voltage input. Hence, there is a possibility to control the current flow in semiconductors , so they don’t just relax and let the current pass by, they can control it, and that is why you can design logic circuits with it.

But as the temperature increases free electrons from valence band of insulators can jump to conduction band and can cause a little conductivity, and then the insulator will act as a semi-conductor.

With respect to a right handed Cartesian coordinate system and given that . A = 4i + k and B = 2i + j _ 3k find A cross B

Answers

Using the left-hand rule,

[tex](4\,\vec\imath+\vec k)\times(2\,\vec\imath+\vec\jmath-3\,\vec k) = \begin{vmatrix}\vec\imath&\vec\jmath&\vec k\\4&0&1\\2&1&-3\end{vmatrix} = -\vec\imath+14\,\vec\jmath+4\,\vec k[/tex]

Then in the right-handed rectangular coordinates, the cross product is the negative of this,

[tex]\boxed{\vec\imath-14\,\vec\jmath-4\,\vec k}[/tex]

A rocket explodes into two fragments, one 25 times heavier than the other. The magnitude of the momentum change of the lighter fragment is A) 25 times as great as the momentum change of the heavier fragment. B) The same as the momentum change of the heavier fragment. C) 1/25 as great as the momentum change of the heavier fragment. D) 5 times as great as the momentum change of the heavier fragment. E) 1/4 as great as the momentum change of the heavier fragment.

Answers

Answer:

B) The same as the momentum change of the heavier fragment.

Explanation:

Since the initial momentum of the system is zero, we have

0 = p + p' where p = momentum of lighter fragment = mv where m = mass of lighter fragment, v = velocity of lighter fragment, and p' = momentum of heavier fragment = m'v' where m = mass of heavier fragment = 25m and v = velocity of heavier fragment.

0 = p + p'

p = -p'

Since the initial momentum of each fragment is zero, the momentum change of lighter fragment Δp = final momentum - initial momentum = p - 0  = p

The momentum change of heavier fragment Δp' = final momentum - initial momentum = p' - 0 = p' - 0 = p'

Since p = -p' and Δp = p and Δp' = -p = p ⇒ Δp = Δp'

So, the magnitude of the momentum change of the lighter fragment is the same as that of the heavier fragment.  

So, option B is the answer

why is nut-cracker 2nd class lever?​

Answers

2nd class leaver refers to such leaver in which load lies between effort and fulcrum.In a nut cracker too load is in between effort and fulcrum.Thus, nut cracker is a 2nd class leaver.......

A man is driving a car at speed 25m/s. calculate the distance covered by it in one hour.​

Answers

Answer:

6.94 km/hr

Explanation:

m/s to km/hr -> Multiply by 18/5

25/(18/5)

=> 25 x 5/18

=> 125/18 km/hr

=> 6.94 km/hr

Answer: 90,000 m = 90 km

Explanation:

Given information

Time = 1 hour

Speed = 25 m/s

Given expression deducted from the given information

Distance = speed × time

Convert units of time

1 hour = 60 minutes

1 minute = 60 seconds

1 hour = 60 × 60 = 3600 seconds

Substitute values into the expression

Distance = 25 × 3600

Simplify by multiplication

Distance = [tex]\boxed{90,000 m=90km}[/tex]

Hope this helps!! :)

Please let me know if you have any questions

As a roller coaster car crosses the top of a 40-m-diameter loop-the-loop, its apparent weight (the normal force) is the same magnitude as the car's weight. What is the car's speed at the top?

Answers

Answer:

40 because if it is the same weight then there is no weight to make the ride slower so it 40

Explanation:

A ball is launched from the ground with a horizontal speed of 30 m/s and a vertical speed of 30 m/s. How far horizontally will it travel in 2 seconds?
A. 30 m
B. 90 m
C. 45 m
D. 60 m

Answers

Answer:

It will travel Vx * t = 30 m/s * 2 s = 60 m

Distance = velocity x time
= 30m/s x 2 sec
= 60 m/s

Convierta 164 decimetros a hectometros

Answers

Answer:

sinco

Explanation:

In the following experiments, identify the independent and dependent variable.

Answers

Answer:

in what experements

Explanation:

A mass weighing 4 lb stretches a spring 4in. Suppose the mass is given an additional in displacement downwards and then released. Assuming no friction and no external force, the natural frequency W (measured in radians per unit time) for the system is? (Recall that the acceleration due to gravity is 32ft/sec2).
a) None of the other alternatives is correct.
b) W = v2 3
c)w=212
d) w = 4/6
e) w=213

Answers

Answer:

4√6 rad/s

Explanation:

Since the spring is initially stretched a length of x = 4 in when the 4 lb mass is placed on it, since it is in equilibrium, the spring force, F = kx equals the weight of the mass W = mg.

So, W = F

mg = kx where m = mass = 4lb, g = acceleration due to gravity = 32 ft/s², k = spring constant and x = equilibrium displacement of spring = 4 in = 4 in × 1ft /12 in = 1/3 ft

making k the spring constant subject of the formula, we have

k = mg/x

substituting the values of the variables into the equation, we have

k = mg/x  

k = 4 lb × 32 ft/s² ÷ 1/3 ft

k = 32 × 4 × 3

k = 384 lbft²/s²

Now, assuming there is no friction and no external force, we have an undamped system.

So, the natural frequency for an undamped system, ω = √(k/m) where k = spring constant = 384 lbft²/s² and m = mass = 4 lb

So, substituting the values of the variables into the equation, we have

ω = √(k/m)

ω = √(384 lbft²/s² ÷ 4 lb)

ω = √96

ω = √(16 × 6)

ω = √16 × √6

ω = 4√6 rad/s

A wire, 0.60 m in length, is carrying a current of 2.0 A and is placed at a certain angle with respect to the magnetic field of strength 0.30 T. If the wire experiences a force of 0.18 N, what angle does the wire make with respect to the magnetic field

Answers

Answer:

[tex]\theta=30 \textdegree[/tex]

Explanation:

From the question we are told that:

Current [tex]I=2.0A[/tex]

Length [tex]L=0.60m[/tex]

Magnetic field [tex]B=0.30T[/tex]

Force [tex]F=0.18N[/tex]

Generally the equation for Force is mathematically given by

[tex]F = BIL sin\theta[/tex]

[tex]sin\theta=\frac{F}{BIL}[/tex]

[tex]\theta=sin^{-1}\frac{0.18}{0.3*2*0.6}[/tex]

[tex]\theta=30 \textdegree[/tex]

What distance do I cover if I travel at 10 m/s E for 10s?

Answers

Answer:

100m

Explanation:

i think this is the answer because the formula for distance is

d=speed×time in this case the speed is 10m/s and the time is 10s therefore the distance will be

10m/s×10s

=100m

I hope this helps

Answer:

100 m

Explanation: this is when you need to find velocity and the formula for velocity is displacement by time taken.

What can light and sound both vary in which one of these ?

Volume
Intensity
Number or prespective?

Answers

Answer:

intensity

Explanation:

b. example, The sun is very bright and intense. and sounds can be very loud.

Read the following sentence from the article. Life can evolve into complex and specialized forms that exploit every possible niche in their surroundings. What is the definition of "exploit" as it is used in this sentence? A to turn to advantage B to invent a new application C to draw on an earlier experience D to narrow down options​

Answers

Answer:

C. To draw on an earlier experience.

it is easier to drag a stone than to kick it?why.​

Answers

Answer:

you are going to expend energy to give a lot of velocity (and momentum) to your foot in order to transfer it the stone air drag this time the kicking speed is for superior to walking speed.

 

                                                  Thank You

What about Iceland's location makes it particularly well-suited to produce electricity from geothermal energy

Answers

The volcanoes in Iceland generate geothermal energy, heating & help with the making of electricity

Answer:

Iceland lies on a boundary where two plates are moving away from each other. Heat from Earth’s interior rises through this plate boundary at a fast rate. This fact makes Iceland well-suited to producing electricity using its abundance of geothermal energy.

Explanation:

Edmentum sample answer.

Other Questions
Find the solution of the differential equation that satisfies the given initial condition. (dP)/(dt) Which expressions have the same solution as 4(6)? Select three options.. 4( 6)4( 6)6( 4)8(2)12( 2) Express sin O as a fraction in simplest form Find the area of circle x that has a radius at coordinates X = (0, 3) and Y = (-3, -1). Round to the nearest tenth. Fix this grammatically incorrect sentence:The boats sails are very colorful. (There is ONLY ONE answer)A. The boats sails are very colorful.B. The boats sails are very colorful.C. The boats sails are very colorful.D. The boat sails are very colorful.E. The sails of the boat are colorful. what is the proportion 4/y= 5/10 Evaluate double integral of f(u,v)=dudv over region R where R is bounded by v^2-nu=0 , v^2-(n+1)u=0 and uv=n,uv=(n+1). Sketch neat graph and shade the bounded region. Clearly mention the points of intersection. Reverse the order of integration then evaluate. Where n is 100. How do you graph this helppp and explain Motivated by the desire to live an idyllic life, people who are part of ______ movements may retreat from the larger society and form their own communities with like-minded others. Trevor is a habitual criminal offender. He has committed dozens of robberies and hundreds of burglaries, and has stolen approximately 30 vehicles. Trevor was 14 when he was first arrested for stealing a car. He is now 44 years old and just got out of prison. He spent 10 years in prison for an armed robbery. According to the ______________ effect, while in prison for 10 years, Trevor was prevented from committing further offenses. how does the statue of liberty and bill of rights relate ? Explain What contribution did the Tuskegee Airmen make in World War II? How do I simplify rational expressions Rosena bought an 8 pack of donuts to share with her friend at school. The donuts cost $28.00 all together. What is the unit rate for the donuts in $/donuts? The man sat patiently while his wife got her nails done.What sentence type is this?Compound-complexSimpleCompoundO ComplexHELP ME PLSSSS Martha Wembly has a history of diabetes mellitus, type 2. How is this different from diabetes mellitus, type 1 What is photosynthesis?? Does the expression represents a positive or negative 3(-2 1/3) Which poetic devices help to create rhythm in a poem? What element has 9 electrons, and 11 neutrons?